Solve f'(x) = 0: Why is There No Solution?

  • Thread starter Thread starter nesan
  • Start date Start date
Click For Summary
The discussion centers on solving the derivative f'(x) = 0 for the function f(x) = (ln x + 2x)^(1/3). A proposed solution of x = -1/2 is invalid because the natural logarithm, ln(x), is only defined for positive x values. Although Wolfram Alpha provides a solution, it uses a definition of logarithms that includes complex numbers, which is not applicable in this context. The key takeaway is that while computer software can yield solutions, they may not align with the constraints of the problem at hand. Thus, the textbook's assertion of "no solution" is correct within the realm of real numbers.
nesan
Messages
74
Reaction score
0

Homework Statement



http://www.wolframalpha.com/input/?i=((1+/+x)+++2)+/+(3(2x+++lnx))^(2/3)+=+0



Solve the equation f'(x) = 0

f(x) = (ln x + 2x)^(1/3)


The Attempt at a Solution



As you can see, for the function

((1 / x) + 2) / (3(2x + lnx))^(2/3) = 0

I can get a value for x which is - 1 / 2

But my textbook says "no solution"

Why is that?
 
Physics news on Phys.org
Mark44 said:
x must be positive in order for ln(x) to be defined.

Oh wow, I totally missed that. Thank you very much. :)
 
That does not mean that wolfram alpha is wrong however. Wolfram alpha uses a definition of the logarithm which is also defined for negative numbers. This definition uses complex numbers.

However, you likely did not encounter complex numbers and complex logarithms yet, so in your problem x=-1/2 is not a valid solution as the logarithm is not well-defined.

This is a bit the danger of using computer software. Their solutions are not wrong, but they can be different solutions from what you want.
 
Question: A clock's minute hand has length 4 and its hour hand has length 3. What is the distance between the tips at the moment when it is increasing most rapidly?(Putnam Exam Question) Answer: Making assumption that both the hands moves at constant angular velocities, the answer is ## \sqrt{7} .## But don't you think this assumption is somewhat doubtful and wrong?

Similar threads

  • · Replies 1 ·
Replies
1
Views
1K
  • · Replies 9 ·
Replies
9
Views
2K
  • · Replies 34 ·
2
Replies
34
Views
3K
  • · Replies 4 ·
Replies
4
Views
1K
Replies
3
Views
2K
  • · Replies 7 ·
Replies
7
Views
2K
  • · Replies 4 ·
Replies
4
Views
2K
  • · Replies 2 ·
Replies
2
Views
2K
  • · Replies 3 ·
Replies
3
Views
2K
Replies
2
Views
3K